Những câu hỏi liên quan
hh hh
Xem chi tiết
nguyễn thùy linh
21 tháng 2 2017 lúc 21:26

x,y,z là số thực à khó đấy số dương thì mk còn làm đc 

chứ số thực mk chịu

Bình luận (0)
NGUYỄN THẾ HIỆP
21 tháng 2 2017 lúc 23:08

Biến đổi tương đương ta CM được BĐT sau: \(x^3+y^3\ge xy\left(x+y\right)\)

Ta có: \(\frac{1}{x^3+y^3+1}\le\frac{1}{xy\left(x+y\right)+xyz}=\frac{1}{xy\left(x+y+z\right)}=\frac{z}{xyz\left(x+y+z\right)}\)

CM tương tự với các phân thức còn lại

Cộng vế theo vế các BĐT đó ta được:

\(A\le\frac{x+y+z}{xyz\left(x+y+z\right)}=\frac{1}{xyz}=1\)

Vậy Max A=1 <=> x=y=z=1

Bình luận (0)
nguyen van giang
Xem chi tiết
Thắng Nguyễn
18 tháng 9 2016 lúc 23:07

Bài 1: \(T=\sqrt{\frac{x^3}{x^3+8y^3}}+\sqrt{\frac{4y^3}{y^3+\left(x+y\right)^3}}\)

\(=\frac{x^2}{\sqrt{x\left(x^3+8y^3\right)}}+\frac{2y^2}{\sqrt{y\left[y^3+\left(x+y\right)^3\right]}}\)

\(=\frac{x^2}{\sqrt{\left(x^2+2xy\right)\left(x^2-2xy+4y^2\right)}}+\frac{2y^2}{\sqrt{\left(xy+2y^2\right)\left(x^2+xy+y^2\right)}}\)

\(\ge\frac{2x^2}{2x^2+4y^2}+\frac{4y^2}{2y^2+\left(x+y\right)^2}\ge\frac{2x^2}{2x^2+4y^2}+\frac{4y^2}{2x^2+4y^2}=1\)

\(\Rightarrow T\ge1\)

Bài 2:

[Toán 10] Bất đẳng thức | Page 5 | HOCMAI Forum - Cộng đồng học sinh Việt Nam

Bình luận (0)
Bích Dao
Xem chi tiết
phan tuấn anh
Xem chi tiết
Hải Đậu Thị
18 tháng 12 2015 lúc 21:26

bài này dễ nhưng bạn phải chứng minh bđt này đã:

\(\frac{1}{a+b+c+d}\le\frac{1}{16}\left(\frac{1}{a}+\frac{1}{b}+\frac{1}{c}+\frac{1}{d}\right)\)

với a;b;c;d là các số dương

bạn có thể cm bđt trên bằng cách biến đổi tương đương hoặc cm bđt Schwat (Sơ-vác)

Mình là 1 phần tử đại diện còn lại là hoàn toàn tt nhé 

ta có \(\frac{1}{3\sqrt{x}+3\sqrt{y}+2\sqrt{z}}=\frac{1}{2\left(\sqrt{x}+\sqrt{y}\right)+\left(\sqrt{y}+\sqrt{z}\right)+\left(\sqrt{x}+\sqrt{z}\right)}\)

\(\le\frac{1}{16}\left(\frac{1}{\sqrt{x}+\sqrt{y}}+\frac{1}{\sqrt{x}+\sqrt{y}}+\frac{1}{\sqrt{y}+\sqrt{z}}+\frac{1}{\sqrt{x}+\sqrt{z}}\right)\)

Tương tự ta cm được 

\(VT\le\frac{1}{16}.4\left(\frac{1}{\sqrt{x}+\sqrt{y}}+\frac{1}{\sqrt{y}+\sqrt{z}}+\frac{1}{\sqrt{z}+\sqrt{x}}\right)\)\(=\frac{1}{4}.3=\frac{3}{4}\)

dấu "=" khi x=y=z

 

 

 

Bình luận (0)
Kiệt Nguyễn Văn
Xem chi tiết
Nguyễn Tất Đạt
2 tháng 9 2018 lúc 16:26

Ta đi c/m BĐT sau: \(x^3+y^3\ge xy\left(x+y\right)\) (*)

Thật vậy (*) \(\Leftrightarrow x^3+y^3-x^2y-xy^2\ge0\)

\(\Leftrightarrow x^2\left(x-y\right)+y^2\left(y-x\right)\ge0\)

\(\Leftrightarrow\left(x-y\right)\left(x^2-y^2\right)\ge0\)

\(\Leftrightarrow\left(x-y\right)^2\left(x+y\right)\ge0\)(luôn đúng)

Áp dụng vào bài toán: 

\(\frac{1}{x^3+y^3+1}\le\frac{1}{xy\left(x+y\right)+1}=\frac{1}{xy\left(x+y+z\right)}\)(Do xyz=1)

Tương tự: \(\frac{1}{y^3+z^3+1}\le\frac{1}{yz\left(x+y+z\right)};\frac{1}{z^3+x^3+1}\le\frac{1}{zx\left(x+y+z\right)}\)

\(\Rightarrow A\le\frac{1}{xy\left(x+y+z\right)}+\frac{1}{yz\left(x+y+z\right)}+\frac{1}{zx\left(x+y+z\right)}=\frac{x+y+z}{xyz\left(x+y+z\right)}=1\)

Vậy Max A = 1. Dấu "=" xảy ra <=> x=y=z=1.

Bình luận (0)
Trầnnhy
Xem chi tiết
Akai Haruma
31 tháng 1 2017 lúc 17:58

Lời giải:

Sử dụng bổ đề: Với \(a,b>0\Rightarrow a^3+b^3\geq ab(a+b)\)

BĐT đúng vì nó tương đương với \((a-b)^2(a+b)\geq 0\) (luôn đúng)

Áp dụng vào bài toán:

\(P\leq \frac{1}{x^3yz(y+z)+1}+\frac{1}{y^3xz(x+z)+1}+\frac{1}{z^3xy(x+y)+1}\)

\(\Leftrightarrow P\leq \frac{1}{x^2(y+z)+xyz}+\frac{1}{y^2(x+z)+xyz}+\frac{1}{z^2(x+y)+xyz}\)

\(\Leftrightarrow P\leq \frac{1}{x(xy+yz+xz)}+\frac{1}{y(xy+yz+xz)}+\frac{1}{z(xy+yz+xz)}=\frac{xy+yz+xz}{xy+yz+xz}=1\)

Vậy \(P_{\max}=1\Leftrightarrow x=y=z=1\)

Bình luận (0)
Nguyễn Thị Ngọc Hân
Xem chi tiết
Akai Haruma
5 tháng 3 2020 lúc 22:11

Lời giải:

Đề cần bổ sung điều kiện $x,y,z>0$

Xét hiệu:

$x^3+y^3-xy(x+y)=(x-y)^2(x+y)\geq 0, \forall x,y>0$

$\Rightarrow x^3+y^3\geq xy(x+y)$

$\Rightarrow x^3+y^3+1=x^3+y^3+xyz\geq xy(x+y+z)$

$\Rightarrow \frac{1}{x^3+y^3+1}\leq \frac{1}{xy(x+y+z)}=\frac{xyz}{xy(x+y+z)}=\frac{z}{x+y+z}$

Hoàn toàn tương tự:

$\frac{1}{y^3+z^3+1}\leq \frac{x}{x+y+z}; \frac{1}{z^3+x^3+1}\leq \frac{y}{x+y+z}$

Cộng theo vế các BĐT trên thu được:

$A\leq \frac{x+y+z}{x+y+z}=1$

Vậy $A_{\max}=1$ khi $x=y=z=1$

Bình luận (0)
 Khách vãng lai đã xóa
toan bai kho
Xem chi tiết
Hoàng Bảo Trân
Xem chi tiết